Homomorphism and relative order questions












1












$begingroup$


1) I have to prove that if $exists$ a nontrivial homomorphism $phi:Arightarrow B$, where A and B are finite and Abelian, then $|A|$ and $|B|$ are not relatively prime.



I know that $phi(A)$ is a subgroup of $B$ and $because$ $B$ is Abelian $phi(A)triangleleft B$. So I think $phi(A) | |B|$ but I can't draw a connection between $|A|$ and $|B|$. Figured this out now. $A/Ker(phi) cong phi(A)$, by Lagrange's, $|A/Ker(phi)|$ divides $|A|$ and hence $|phi(A)|$ divides $|A|$. Therefore, $|A|$ and $|B|$ are not relatively prime.



2) Deduce that $exists$ a nontrivial homomorphism $Brightarrow A$

Do we have to prove the converse of (1) because the orders are relatively prime? Figured it out. We can use the same argument as above.










share|cite|improve this question











$endgroup$












  • $begingroup$
    (1): Show that the order of $phi(A)$ is a divisor of $|A|$.
    $endgroup$
    – Arturo Magidin
    Dec 3 '18 at 20:29










  • $begingroup$
    Made an edit, able to show for elements of A but how can I do that for the whole group A?
    $endgroup$
    – manifolded
    Dec 3 '18 at 20:53










  • $begingroup$
    You can use the isomorphism theorems and Lagrange’s Theorem.
    $endgroup$
    – Arturo Magidin
    Dec 3 '18 at 20:55










  • $begingroup$
    And, no, the fact that The order of $phi(a)$ divides the order of $a$ for each $a$ does not “lead” to $|phi(A)|$ dividing $|A|$. It’s actually the other way around.
    $endgroup$
    – Arturo Magidin
    Dec 3 '18 at 21:02












  • $begingroup$
    True, was able to show $|A/N|$ divides $|A|$ and $A/N cong phi(A)$ and hence $phi(A)$ dividies $|A|$. Thanks for the hint. How about part (2)?
    $endgroup$
    – manifolded
    Dec 3 '18 at 21:06
















1












$begingroup$


1) I have to prove that if $exists$ a nontrivial homomorphism $phi:Arightarrow B$, where A and B are finite and Abelian, then $|A|$ and $|B|$ are not relatively prime.



I know that $phi(A)$ is a subgroup of $B$ and $because$ $B$ is Abelian $phi(A)triangleleft B$. So I think $phi(A) | |B|$ but I can't draw a connection between $|A|$ and $|B|$. Figured this out now. $A/Ker(phi) cong phi(A)$, by Lagrange's, $|A/Ker(phi)|$ divides $|A|$ and hence $|phi(A)|$ divides $|A|$. Therefore, $|A|$ and $|B|$ are not relatively prime.



2) Deduce that $exists$ a nontrivial homomorphism $Brightarrow A$

Do we have to prove the converse of (1) because the orders are relatively prime? Figured it out. We can use the same argument as above.










share|cite|improve this question











$endgroup$












  • $begingroup$
    (1): Show that the order of $phi(A)$ is a divisor of $|A|$.
    $endgroup$
    – Arturo Magidin
    Dec 3 '18 at 20:29










  • $begingroup$
    Made an edit, able to show for elements of A but how can I do that for the whole group A?
    $endgroup$
    – manifolded
    Dec 3 '18 at 20:53










  • $begingroup$
    You can use the isomorphism theorems and Lagrange’s Theorem.
    $endgroup$
    – Arturo Magidin
    Dec 3 '18 at 20:55










  • $begingroup$
    And, no, the fact that The order of $phi(a)$ divides the order of $a$ for each $a$ does not “lead” to $|phi(A)|$ dividing $|A|$. It’s actually the other way around.
    $endgroup$
    – Arturo Magidin
    Dec 3 '18 at 21:02












  • $begingroup$
    True, was able to show $|A/N|$ divides $|A|$ and $A/N cong phi(A)$ and hence $phi(A)$ dividies $|A|$. Thanks for the hint. How about part (2)?
    $endgroup$
    – manifolded
    Dec 3 '18 at 21:06














1












1








1





$begingroup$


1) I have to prove that if $exists$ a nontrivial homomorphism $phi:Arightarrow B$, where A and B are finite and Abelian, then $|A|$ and $|B|$ are not relatively prime.



I know that $phi(A)$ is a subgroup of $B$ and $because$ $B$ is Abelian $phi(A)triangleleft B$. So I think $phi(A) | |B|$ but I can't draw a connection between $|A|$ and $|B|$. Figured this out now. $A/Ker(phi) cong phi(A)$, by Lagrange's, $|A/Ker(phi)|$ divides $|A|$ and hence $|phi(A)|$ divides $|A|$. Therefore, $|A|$ and $|B|$ are not relatively prime.



2) Deduce that $exists$ a nontrivial homomorphism $Brightarrow A$

Do we have to prove the converse of (1) because the orders are relatively prime? Figured it out. We can use the same argument as above.










share|cite|improve this question











$endgroup$




1) I have to prove that if $exists$ a nontrivial homomorphism $phi:Arightarrow B$, where A and B are finite and Abelian, then $|A|$ and $|B|$ are not relatively prime.



I know that $phi(A)$ is a subgroup of $B$ and $because$ $B$ is Abelian $phi(A)triangleleft B$. So I think $phi(A) | |B|$ but I can't draw a connection between $|A|$ and $|B|$. Figured this out now. $A/Ker(phi) cong phi(A)$, by Lagrange's, $|A/Ker(phi)|$ divides $|A|$ and hence $|phi(A)|$ divides $|A|$. Therefore, $|A|$ and $|B|$ are not relatively prime.



2) Deduce that $exists$ a nontrivial homomorphism $Brightarrow A$

Do we have to prove the converse of (1) because the orders are relatively prime? Figured it out. We can use the same argument as above.







abstract-algebra group-theory finite-groups abelian-groups group-homomorphism






share|cite|improve this question















share|cite|improve this question













share|cite|improve this question




share|cite|improve this question








edited Dec 3 '18 at 21:35







manifolded

















asked Dec 3 '18 at 20:19









manifoldedmanifolded

736




736












  • $begingroup$
    (1): Show that the order of $phi(A)$ is a divisor of $|A|$.
    $endgroup$
    – Arturo Magidin
    Dec 3 '18 at 20:29










  • $begingroup$
    Made an edit, able to show for elements of A but how can I do that for the whole group A?
    $endgroup$
    – manifolded
    Dec 3 '18 at 20:53










  • $begingroup$
    You can use the isomorphism theorems and Lagrange’s Theorem.
    $endgroup$
    – Arturo Magidin
    Dec 3 '18 at 20:55










  • $begingroup$
    And, no, the fact that The order of $phi(a)$ divides the order of $a$ for each $a$ does not “lead” to $|phi(A)|$ dividing $|A|$. It’s actually the other way around.
    $endgroup$
    – Arturo Magidin
    Dec 3 '18 at 21:02












  • $begingroup$
    True, was able to show $|A/N|$ divides $|A|$ and $A/N cong phi(A)$ and hence $phi(A)$ dividies $|A|$. Thanks for the hint. How about part (2)?
    $endgroup$
    – manifolded
    Dec 3 '18 at 21:06


















  • $begingroup$
    (1): Show that the order of $phi(A)$ is a divisor of $|A|$.
    $endgroup$
    – Arturo Magidin
    Dec 3 '18 at 20:29










  • $begingroup$
    Made an edit, able to show for elements of A but how can I do that for the whole group A?
    $endgroup$
    – manifolded
    Dec 3 '18 at 20:53










  • $begingroup$
    You can use the isomorphism theorems and Lagrange’s Theorem.
    $endgroup$
    – Arturo Magidin
    Dec 3 '18 at 20:55










  • $begingroup$
    And, no, the fact that The order of $phi(a)$ divides the order of $a$ for each $a$ does not “lead” to $|phi(A)|$ dividing $|A|$. It’s actually the other way around.
    $endgroup$
    – Arturo Magidin
    Dec 3 '18 at 21:02












  • $begingroup$
    True, was able to show $|A/N|$ divides $|A|$ and $A/N cong phi(A)$ and hence $phi(A)$ dividies $|A|$. Thanks for the hint. How about part (2)?
    $endgroup$
    – manifolded
    Dec 3 '18 at 21:06
















$begingroup$
(1): Show that the order of $phi(A)$ is a divisor of $|A|$.
$endgroup$
– Arturo Magidin
Dec 3 '18 at 20:29




$begingroup$
(1): Show that the order of $phi(A)$ is a divisor of $|A|$.
$endgroup$
– Arturo Magidin
Dec 3 '18 at 20:29












$begingroup$
Made an edit, able to show for elements of A but how can I do that for the whole group A?
$endgroup$
– manifolded
Dec 3 '18 at 20:53




$begingroup$
Made an edit, able to show for elements of A but how can I do that for the whole group A?
$endgroup$
– manifolded
Dec 3 '18 at 20:53












$begingroup$
You can use the isomorphism theorems and Lagrange’s Theorem.
$endgroup$
– Arturo Magidin
Dec 3 '18 at 20:55




$begingroup$
You can use the isomorphism theorems and Lagrange’s Theorem.
$endgroup$
– Arturo Magidin
Dec 3 '18 at 20:55












$begingroup$
And, no, the fact that The order of $phi(a)$ divides the order of $a$ for each $a$ does not “lead” to $|phi(A)|$ dividing $|A|$. It’s actually the other way around.
$endgroup$
– Arturo Magidin
Dec 3 '18 at 21:02






$begingroup$
And, no, the fact that The order of $phi(a)$ divides the order of $a$ for each $a$ does not “lead” to $|phi(A)|$ dividing $|A|$. It’s actually the other way around.
$endgroup$
– Arturo Magidin
Dec 3 '18 at 21:02














$begingroup$
True, was able to show $|A/N|$ divides $|A|$ and $A/N cong phi(A)$ and hence $phi(A)$ dividies $|A|$. Thanks for the hint. How about part (2)?
$endgroup$
– manifolded
Dec 3 '18 at 21:06




$begingroup$
True, was able to show $|A/N|$ divides $|A|$ and $A/N cong phi(A)$ and hence $phi(A)$ dividies $|A|$. Thanks for the hint. How about part (2)?
$endgroup$
– manifolded
Dec 3 '18 at 21:06










0






active

oldest

votes











Your Answer





StackExchange.ifUsing("editor", function () {
return StackExchange.using("mathjaxEditing", function () {
StackExchange.MarkdownEditor.creationCallbacks.add(function (editor, postfix) {
StackExchange.mathjaxEditing.prepareWmdForMathJax(editor, postfix, [["$", "$"], ["\\(","\\)"]]);
});
});
}, "mathjax-editing");

StackExchange.ready(function() {
var channelOptions = {
tags: "".split(" "),
id: "69"
};
initTagRenderer("".split(" "), "".split(" "), channelOptions);

StackExchange.using("externalEditor", function() {
// Have to fire editor after snippets, if snippets enabled
if (StackExchange.settings.snippets.snippetsEnabled) {
StackExchange.using("snippets", function() {
createEditor();
});
}
else {
createEditor();
}
});

function createEditor() {
StackExchange.prepareEditor({
heartbeatType: 'answer',
autoActivateHeartbeat: false,
convertImagesToLinks: true,
noModals: true,
showLowRepImageUploadWarning: true,
reputationToPostImages: 10,
bindNavPrevention: true,
postfix: "",
imageUploader: {
brandingHtml: "Powered by u003ca class="icon-imgur-white" href="https://imgur.com/"u003eu003c/au003e",
contentPolicyHtml: "User contributions licensed under u003ca href="https://creativecommons.org/licenses/by-sa/3.0/"u003ecc by-sa 3.0 with attribution requiredu003c/au003e u003ca href="https://stackoverflow.com/legal/content-policy"u003e(content policy)u003c/au003e",
allowUrls: true
},
noCode: true, onDemand: true,
discardSelector: ".discard-answer"
,immediatelyShowMarkdownHelp:true
});


}
});














draft saved

draft discarded


















StackExchange.ready(
function () {
StackExchange.openid.initPostLogin('.new-post-login', 'https%3a%2f%2fmath.stackexchange.com%2fquestions%2f3024604%2fhomomorphism-and-relative-order-questions%23new-answer', 'question_page');
}
);

Post as a guest















Required, but never shown

























0






active

oldest

votes








0






active

oldest

votes









active

oldest

votes






active

oldest

votes
















draft saved

draft discarded




















































Thanks for contributing an answer to Mathematics Stack Exchange!


  • Please be sure to answer the question. Provide details and share your research!

But avoid



  • Asking for help, clarification, or responding to other answers.

  • Making statements based on opinion; back them up with references or personal experience.


Use MathJax to format equations. MathJax reference.


To learn more, see our tips on writing great answers.




draft saved


draft discarded














StackExchange.ready(
function () {
StackExchange.openid.initPostLogin('.new-post-login', 'https%3a%2f%2fmath.stackexchange.com%2fquestions%2f3024604%2fhomomorphism-and-relative-order-questions%23new-answer', 'question_page');
}
);

Post as a guest















Required, but never shown





















































Required, but never shown














Required, but never shown












Required, but never shown







Required, but never shown

































Required, but never shown














Required, but never shown












Required, but never shown







Required, but never shown







Popular posts from this blog

Plaza Victoria

In PowerPoint, is there a keyboard shortcut for bulleted / numbered list?

How to put 3 figures in Latex with 2 figures side by side and 1 below these side by side images but in...